Which statement correctly compares the centers of the distributions?
A. The median penguin height is greater at Park Zoo than at Cityview Zoo.
B. The median penguin heights are the same.
C. The median penguin height is greater at Cityview Zoo than at Park
Zoo.
D. The range of penguin heights is greater at Cityview Zoo than at
Park Zoo.

Which Statement Correctly Compares The Centers Of The Distributions?A. The Median Penguin Height Is Greater

Answers

Answer 1
The correct answer to this question is c
Answer 2

The median penguin height is greater at Cityview Zoo than at Park

Zoo, Option C is correct.

Mode is the most occuring number.

The range is the difference of the highest value and the lowest value.

The median is the middle value in a set of data

After finding the range and medians of the given data.

The median penguin at Cityview Zoo is 42 cm tall,

The median penguin at Park Zoo is barely 41 cm tall.

Cityview Zoo's median penguin height is higher than that of Park Zoo.

Hence, the median penguin height is greater at Cityview Zoo than at Park Zoo.

To learn more on Statistics click:

https://brainly.com/question/30218856

#SPJ7


Related Questions

Help me with this answer I don’t it

Answers

Answer:

f(-2) = g(-2) this is the answer

Darcy gave her hairstylist a $ 4.90 The tip was 14​% of the cost of the haircut . Write an equation to find​ b, the cost of the haircut.

Answers

Answer:

Equation: 4.90/b = 14/100

Solution: b = $35

Step-by-step explanation:

Variable b = cost of the haircut

Solve for b:

4.90/b = 14/100

490 = 14b

35 = b

Check your work:

35 × 0.14 = 4.90

Correct!

Simplify:......................................................

Answers

Answer:

...

Step-by-step explanation:...

The cut off number is 2x-1

Which rules of exponents will be used to evallate this expression? Select three options.

Answers

I need the answer choices and the expression you’re referring to

international system of 89643092 in words​

Answers

Answer:

Eighty nine million six hundred forty three thousand ninety two

Step-by-step explanation:

89,643,092

=> Eighty nine million six hundred forty three thousand ninety two

The probability distribution for a random variable x is given in the table X: -5,-3,-2,0,2,3 Probability: .17,.13,.33,.16,.11,.10 Find the probability that X <_-3

Answers

Answer:

0.3 probability that [tex]x \leq -3[/tex]

Step-by-step explanation:

The probability distribution is given in the table.

Probability that x <= -3

The values that are -3 or lower are -3 and -5. So

[tex]P(X \leq -3) = P(X = -3) + P(X = -5)[/tex]

From the table:

[tex]P(X = -3) = 0.13, P(X = -5) = 0.17[/tex]. So

[tex]P(X \leq -3) = P(X = -3) + P(X = -5) = 0.13 + 0.17 = 0.3[/tex]

0.3 probability that [tex]x \leq -3[/tex]

Answer:0.3

Step-by-step explanation:

Determine whether the following event is mutually exclusive or not mutually exclusive.

Choosing a student who is a mathematics major or a business major from a nearby university to participate in a research study. (Assume that each student only has one major.)

Answers

Answer:

The event is mutually exclusive.

Step-by-step explanation:

Mutually exclusive events are events that cannot exist simultaneously.

Thus, events that are not mutually exclusive can exist simultaneously.

Since each student only has one major, a single student cannot be both a mathematics major and a business major.

So, the event is mutually exclusive.

Find the area of the following shape:

Answers

Answer:

36cm^2

Step-by-step explanation:

total area: 6x(4+3)=42

total area excluding the space: 42-(2x3)=36

Answer:

36 cm squared

Step-by-step explanation:

To solve this problem, I first construct a line. (shown in yellow in the first photo)

I then find the area of the top rectangle. (6 cm * 4 cm = 24 cm squared.)

Next, I find the area of the lower rectangle. But...to do that I have to find the length of the line that I constructed. To do this, I do  6cm-2cm=4cm.

Then I can find the area of the lower rectangle. (4cm*3cm=12cm squared.)

add up the area of both of the rectangles and.........12+24=36 cm squared

I need help I’ll give u brainlest

Answers

Answer:

[tex]V=280[/tex] cubic inches

Step-by-step explanation:

Volume formula for triangular prism is  [tex]V=\frac{1}{2} bhl[/tex]

[tex]V=\frac{1}{2} (7)(10)(8)[/tex]

[tex]V=\frac{1}{2}(560)[/tex]

[tex]V=280[/tex]

Hope this helps

Which number's estimate written as a single digit times a power of 10 will have a negative exponent?

Answers

105 i hope this helps if not then i’m sorry

Please please help me please I really need help please just tap on picture and you will see the question

Answers

Answer:

No , it is not a right angle triangle

Step-by-step explanation:

according to the pythagoras theorem in right angled triangle sum of square of two sides is equal to the square of it's hypotenuse.

using pythagoras theorem

a^2 + b^2 = c^2

9^2 + 16^2 = 25^2

81 + 256 = 625

337 = 625

since sum of square of two smallest sides of a triangle is not equal to the square of it's hypotenuse it can be concluded that the given figure does not form right angle triangle.

Heyy!! Can someone help me please!!

3 (5x + 2) - 2 (4x -4)

I don’t know what to doooo!!

Answers

Answer:

7x + 14

Step-by-step explanation:

the first thing to do is expand the parentheses/brackets.

3(5x + 2) -2(4x - 4) will be

3(5x) + 3(2) -2(4x) -2(-4)

= 15x + 6 - 8x + 8

collect like terms

15x - 8x + 6 + 8 = 7x + 14

the answer is 7x + 14

Answer:

3(5x+2)-2(4x-4)

15x+6-8x+8

15x-8x+6+8

7x+14

A geneticist conducts an experiment with peas, one sample of offspring consisted of 450 green peas and 157 yellow peas. Based on these results, estimate the probability of getting an offspring pea that is green.

Answers

Answer: 0.738

Step-by-step explanation:

11. Mendelian Genetics. When Mendel conducted his famous genetics experiments with peas, one sample of offspring consisted of 428 green peas and 152 yellow peas. Based on those results, estimate the probability of getting an offspring pea that is green. Is the result reasonably close to the value that was expected?

p0 = 428/(428 + 152) = 0.737931

If you round your answer of 0.737931 to three decimals you will

get 0.738.

A person walks 1/6 mile in 1/18 hour.

The person's speed is _ miles per hour.

Answers

This Is What I Got!

Hope This Helps! :)

Have A Good Day!!

And If You Can I Wouldn't Mind A Brainliest! :))

Answer:

Divide 1/6 miles to 1/12hour since u wanna find our miles per hour

So it’ll be : 1/6 / 1/12

= 1/6 x 12/1

= 2 miles

The diagram shows a right-angled triangle.
xo
26 cm
17 cm
Find the size of angle x.
Give your answer correct to 1 decimal place.

Answers

Answer:

Diagram? I don't see a diagram.

Where is the diagram?

Step-by-step explanation:

11x+7y=17
solve for y

Answers

[tex]\implies {\blue {\boxed {\boxed {\purple {\sf {\: y = \frac{17 - 11x}{7} }}}}}}[/tex]

[tex]\large\mathfrak{{\pmb{\underline{\red{Step-by-step\:explanation}}{\red{:}}}}}[/tex]

[tex]\\11x + 7y = 17[/tex]

[tex] \\➺ \: 7y = 17 - 11x[/tex]

[tex]\\➺ \: y = \frac{17 - 11x}{7} [/tex]

[tex]\bold{ \green{ \star{ \orange{Mystique35}}}}⋆[/tex]

Given points (-3;-6), G(3; -2) and H(6; 1); determine:
(a) The equation of line FG

Answers

Answer:

The equation of line FG is [tex]y = \frac{2}{3}x - 4[/tex]

Step-by-step explanation:

Equation of a line:

The equation of a line has the following format:

[tex]y = mx + b[/tex]

In which m is the slope and b is the y-intercept.

F(-3;-6), G(3; -2)

When we have two points, the slope is given by the change in y divided by the change in x. So

Change in y : -2 - (-6) = -2 + 6 = 4

Change in x: 3 - (-3) = 3 + 3 = 6

Slope: [tex]m = \frac{4}{6} = \frac{2}{3}[/tex]

So

[tex]y = \frac{2}{3}x + b[/tex]

Finding b:

(3; -2) means that when [tex]x = 3, y = -2[/tex]. We use this to find b.

[tex]y = \frac{2}{3}x + b[/tex]

[tex]-2 = \frac{2}{3}(3) + b[/tex]

[tex]2 + b = -2[/tex]

[tex]b = -4[/tex]

The equation of line FG is [tex]y = \frac{2}{3}x - 4[/tex]

How many liters each of a 25% acid solution and a 50% acid solution must be used to produce 80 liters of a 40% acid solution?

Answers

Answer:

32 and 48 liters

Step-by-step explanation:

Let 25% solution is x liters, then 50% solution is (80 - x) liters.

Acid content is going to be same:

0.25x + 0.5(80 - x) = 80*0.40.25x - 0.5x + 40 = 320.25x = 8x = 8/0.25x = 32 liters

So 32 liters of 25% solution and 80 - 32 = 48 liters of 50% solution

Let V be the set of all 3x3 matrices with Real number entries, with the usual definitions of scalar multiplication and vector addition. Consider whether V is a vector space over C. Mark all true statements (there may be more than one).

a. The scalar closure axiom is satisfied
b. The additive inverse axiom is not satisfied
c The additive inverse axiom is satisfied
d. The additive closure axiom is not satisfied
e. The scalar closure axiom is not satisfied
f. The additive closure axiom is satsified
g. V is not a vector space over C
h. V is a vector space over C
i. The zero axiom is satisfied
j. The zero axiom is not satisfied

Answers

Answer:

the Scalar Closure axiom is not satisfiedV is not a Vector Space of CThe Additive Closure axiom is satisfied.

Step-by-step explanation:

According to the Question,

Given That, Let V be the set of all 3x3 matrices with Real number entries, with the usual definitions of scalar multiplication and vector addition. Consider whether V is a vector space over C.For V is a vector space over C and V is Set of 3x3 Matrices with Real entries.

Then, For any u,w ∈ V ⇒ u+w ∈ V

And u∈V and z∈C ⇒ z u ∈ V

So, If we take any z= i ∈ C

and V be any 3x3 matrices with Real entrices.

then, z,v ∉ V  ∴z,v Has Complex entries

So, the Scalar Closure axiom is not satisfied

Hence, V is not a Vector Space of C

Any u,w ∈ W ⇒ u+w ∈ V

So, The Additive Closure axiom is satisfied.

Simplify the expression.

33 · 32 + 12 ÷ 4

Answers

Answer:

1059

Step-by-step explanation:

33 · 32 + 12 ÷ 4

PEMDAS

Multiply and divide first from left to right

1056 + 3

Then add

1059

[tex]\huge\textsf{Hey there!}[/tex]

[tex]\mathsf{33\times32+12\div4}\\\\\mathsf{33\times32= \boxed{\bf 1,056}}\\\\\mathsf{\bold{1,056}+12\div4}\\\\\mathsf{12\div4=\boxed{\bf 3}}\\\\\mathsf{1,056+\bf 3}\\\mathsf{= \boxed{\bf 1,059}}\\\\\\\boxed{\boxed{\large\textsf{Answer: \huge \bf 1,059}}}\huge\checkmark[/tex]

[tex]\large\textsf{Good luck on your assignment and enjoy your day!}[/tex]

~[tex]\frak{Amphitrite40:)}[/tex]

What is the average score of runa 140,96 and 13?​

Answers

[tex]\huge\textsf{Hey there!}[/tex]

[tex]\large\textsf{Formula:}[/tex]

[tex]\mathsf{\dfrac{Sum\ of\ all\ terms}{Total\ number\ of\ values\ in\ the\ data}= your\ mean/average}[/tex]

[tex]\mathsf{\dfrac{140+96+13}{3}}[/tex]

[tex]\mathsf{140+36+13}[/tex]

[tex]\mathsf{140 + 96 = \bf 236}[/tex]

[tex]\mathsf{236 + 13}[/tex]

[tex]\mathsf{ = \bf 249}[/tex]

[tex]\mathsf{\dfrac{249}{3}}[/tex]

[tex]\mathsf{= \bf 83}[/tex]

[tex]\boxed{\boxed{\large\textsf{Possible answer: \huge \bf 83}}}\huge\checkmark[/tex]

[tex]\large\textsf{Good luck on your assignment and enjoy your your day!}[/tex]

~[tex]\frak{Amphitrite1040:)}[/tex]

2 divided by 0.75 full divison work i dont just need the answer​

Answers

Answer:

0.375

Step-by-step explanation:

Check the picture below.

whenever we do division of decimals, we have to mind how many decimals are there on each amount, the dividend as well as the divisor, that way we pad with zeros the other amount accordingly whilst losing the dot, for example, to say divide 3 by 0.123, 3 has no decimals, whilst 0.123 has three decimals, so we can just divide 3000 by 0123, so dividing 3 by 0.123 is the same as dividing 3000 by 123.  Another example, if we were to divide say 23.761 by 555.89331, the dividend has 3 decimals, that means 3 zeros the other way, the divisor has 5 decimals, that means 5 zeros the other way while losing the dots, so we'd end up dividing 2376100000 by 55589331000, which we can simplify to just 2376100 by 5589331, as you can see in the picture in this case.

What is the area of this figure?

Answers

Answer:

90km² only if it is parallelogram

Step-by-step explanation:

base = 9km

height=10km

area of parallelogram = b x h

=9km x 10km

=90km²

Answer:

A = 90km2

Step-by-step explanation:

Area of a rhombus is:

1. A = s x h (if given side and height)

2. A = 1/2 a x b (if given lengths of diagonals)

3. A = s^2 sin A (if given side and length)

Therefore from your problem, height and side is given thus, you'll use number 1

A = s x h

A = 9km x 10km = 90km2

 Marsha has a bag that contains 4 green marbles, 8 yellow marbles , and 20 red marbles . If she chooses one marble from the bag, what is the probability that the marble is not red?

PLEASE HELP IF YOURE GOOD AT GEOMETRY!!

Answers

Answer:

C. 3/8

HOPE THIS HELPS :)

Answer:

c. 3/8

Step-by-step explanation:

first you need the denomerator by adding all marbles together which equals 32. now for the munerator you need the sum of the green and yellow marbles. this equals 12. so your fraction is 12/32. next we simplify. we can divide both numbers by 4. getting us a fraction of 3/8.

Suppose that g(x)= f(x)+ 6. Which statement best compares the graph of g(x) with the graph of f(x)?

A. The graph of g(x) is the graph of f(x) shifted 6 units down.

B. The graph of g(x) is the graph of f(x) shifted to the right.

C. The graph of g(x) is the graph of f(x) shifted 6 units to the left.

D. The graph of g(x) is the graph of f(x) shifted 6 units up.

Answers

Answer:

D

Step-by-step explanation:

The + 6 moves it up 6 units.

The correct answer is (D) "The graph of g(x) is the graph of f(x) shifted 6 units up."

What is the function?

A relationship between a group of inputs and one output is referred to as a function. In plain English, a function is an association between inputs in which each input is connected to precisely one output. A domain, codomain, or range exists for every function. Typically, f(x), where x is the input, is used to represent a function.

When we add a constant to a function, such as in the case of g(x) = f(x) + 6, it will shift the graph of f(x) upward by 6 units.

This is because, for any value of x, the value of f(x) will be added to 6, resulting in a vertical shift of the entire graph.

Option (A) is incorrect because adding 6 to f(x) would shift the graph up, not down.

Option (B) is incorrect because adding a constant to a function does not cause it to shift horizontally.

Option (C) is incorrect because adding 6 to f(x) would shift the graph right, not left.

D. The graph of g(x) is the graph of f(x) shifted 6 units up. Adding a constant term to a function will shift the graph of the function vertically. In this case, adding 6 to f(x) will shift the graph of f(x) upward by 6 units, resulting in the graph of g(x).

Learn more about function here:

https://brainly.com/question/29633660

#SPJ7

he speeds (in MPH) of automobiles traveling in a city are given below:
20, 35, 42, 52, 65, 49, 24, 37, 23, 41, 50, 58
The mean speed of the cars is

Answers

Answer:

Mean speed = 41.3 mph

Step-by-step explanation:

Given that,

The speeds of an automobiles are given below:

20, 35, 42, 52, 65, 49, 24, 37, 23, 41, 50, 58

We need to find the mean speed of the cars.

Mean = sum of observations/ no. of observation

[tex]M=\dfrac{20+35+42+52+65+49+24+37+23+41+50+58}{12}\\\\M=41.3[/tex]

So, the mean speed of the cars is equal to 41.3 mph.

andrea is planning a birthday party and wants to include a cheese board with the desserts.
she reads online that she should have 110g of cheese per person ,but the cheese is sold in blocks of 500g
How many blocks of cheese should she buy to ensure that each guest can have 110g of cheese?​

Answers

Step-by-step explanation:

how many people in the party please ?

If f(x) = x -2 and g(x) = 2x – 6, then g(4)/f(3) =​

Answers

Answer:

Step-by-step explanation:

(2×4-6)/(3-2)=2

Answer:

[tex]{ \tt{f(x) = x - 2}} \\ { \bf{f(3) = 3 - 2 = 1}} \\ \\ { \tt{g(x) = 2x - 6}} \\ { \bf{g(4) = 2(4) - 6 = 2}} \\ \\ { \boxed{ \tt{ \frac{g(4)}{f(3)} = \frac{2}{1} = 2}}}[/tex]

Joe drives for 3 hours and covers 201 miles. In miles per hour, how fast was he driving?​

Answers

Answer:

67 mph

Step-by-step explanation:

201/3 = 67

HELP HELP HELP
Solve this

Answers

Answer:

What is the cos theta for, i would use sin to solve for theta and then we would get 41.25 degrees.

Step-by-step explanation:

Other Questions
A cell phone company wants to determine the average amount of data that their smart phone customers use each month. Which group would best represent a sample of the population?one thousand of their customers with the most expensive monthly plansone thousand of their customers who most recently signed up with the companyone thousand of their customers with a current smart phone data planone thousand of their customers who called in for technical support Complete the expanded form of this number.62,719Enter numbers from greatest to least place value starting on the left. Describe the process of how the AmericanEconomy Responded to the War Effort and thetransformation from a peacetime to wartimeeconomy? Which of the following is a benefit of doing business with an online bank?A. Greater variety of servicesB. Personalized service C. Variety of deposit locationsOD. Typically higher savings rates Two examples of medicalization effecting how we experience our bodies What is needed to Run A Brushless DC motor I got a question. Im writing an essay and its typically intro 3 paragraphs and conclusion. Which is what I was doing. But Im on my second paragraph and Im talking about 2 completely different things to avoid making another paragraph. Is it fine to talk about 2 different things in 1 paragraph or should I separate it into 2. A ball is dropped from the top of a building. It initiallymoves at 40 m/s. After 0.5 seconds, it moves at 3.8m/s.What force is responsible for the slowing of the ball?air resistancegravitynormaltension What is the range of g(x) = -2 [X + 3] + 2? An empty container has a capacity of 60000 liters to 1 s.f. Tony pours in 5400 liters of water to 2 s.f. He says he filled more than 10 % of container is he correct ? Show working The oxidase test is another biochemical test used to distinguish aerobic versus anaerobic metabolism in microbes. Research the oxidase test and describe how it is performed. Include examples of oxidase-positive and oxidase-negative microbes in your answer. SOLVE AND NO LINKS x 3 = 5 As countries develop economically from being low-income to higher income, what is the most significant epidemiologic trend?Group of answer choicesa. The burden of disease shifts from communicable disease to infectious diseaseb. None of the abovec. The burden of disease shifts from younger people to older peopled. The share of communicable disease declines and the share of non-communicable disease increases How many joules of heat energy are required to raise the temperature of 100.0 g of aluminum by 120.0C? The specific heat of aluminum is 0.897 J/g.C. 2 3 If someone can pls give the answer with steps that would be greatly appreciated :) Which of the following is a reason not to extract mineral resources from the ground? It can improve technology. It can allow the resource to be used in a helpful way. It can cause air, soil, or water pollution. It can provide new jobs. Match each situation with the method of government intervention used to rectify it. price floor contractionary fiscal policy price ceiling expansionary fiscal policy People have too much money, and there is a danger of inflation. arrowRight The GDP has fallen to an all-time low, and there is low demand for most goods. arrowRight Few farmers produce cotton because profits are at the equilibrium price. arrowRight Prices of staple foods have shot up because of shortages after an earthquake. arrowRight The most recent financial statements for Summer Tyme, Inc., are shown here: Income Statement Balance SheetSales $3,700 Current assets $4,500 Current liabilities $960 Costs 2,400 Fixed assets 5,200 Long-term debt 3,620 Taxable income $1,300 Equity 5,120 Taxes (21%) 273 Total $9,700 Total $9,700 Net income $1,027 Assets, costs, and current liabilities are proportional to sales. Long-term debt and equity are not. The company maintains a constant 60 percent dividend payout ratio. As with every other firm in its industry, next year's sales are projected to increase by exactly 30 percent. Required:What is the external financing needed? (Do not round your intermediate calculations.)EFN = needed new long-term debt and/or external equity PLS HELP ASAP (20 POINTS)What do manufacturing factories make on the East Coast of Florida? yo write Ato Z and 1 to 50